Sauter au contenu

Section 4.8 Le Théorème de Stokes

Cette section s’occupe du théorème de Stokes
 1 
Sir George Gabriel Stokes (1819-1903), physicien et mathématicien irlandais. En plus du théorème de Stokes, il est connu pour les équations de Navier-Stokes utilisées pour l’étude de la dynamique des fluides.
, un analogue du théorème de Green, dans sa version tangentielle, mais en trois dimensions. Le résultat lie l’intégrale d’un champ à travers une surface de \(\R^3\) à une intégrale autour de la frontière de la surface.

Sous-section Le Théorème de Stokes

Remarquons que,
  • dans le théorème de Stokes, \(\cS\) doit être une surface orientée, elle ne peut donc pas être un ruban de Möbius;
  • si \(\cS\) est contenue dans le plan \(Oxy\text{,}\) alors le théorème de Stokes devient la version tangentielle du théorème de Green;
  • si \(\partial \cS\) est une courbe fermée simple et que,
    • lorsqu’on regarde \(\partial \cS\) de la partie positive de l’axe des \(z\text{,}\) elle est orientée dans le sens antihoraire (voir la figure dans le Théorème 4.8.1), alors
    • \(\vn\) pointe vers le haut, c’est-à-dire qu’il a une composante \(\vk\) positive près de \(\partial \cS\text{.}\)
Avant de faire des exemples, faisons quelques remarques.
  • Le théorème de Stokes dit que \(\oint_{\cC}\vF\cdot \dee{\vr} =\iint_{\cS}\vnabla\times\vF\cdot\vn\ \dee{S}\) pour toute surface (convenablement orientée) dont le bord est \(\cC\text{.}\) Ainsi, si \(\cS_1\) et \(\cS_2\) sont deux surfaces différentes (convenablement orientées) qui ont un bord commun \(\cC\text{,}\) alors
    \begin{equation*} \iint_{\cS_1}\vnabla\times\vF\cdot\vn\ \dee{S} =\iint_{\cS_2}\vnabla\times\vF\cdot\vn\ \dee{S}\text{.} \end{equation*}
    Par exemple, si \(\cC\) est le cercle
    \begin{equation*} \cC=\Set{(x,y,z)}{x^2+y^2=1,\ z=0} \end{equation*}
    orienté dans le sens antihoraire, alors
    \begin{align*} \cS_1&= \Set{(x,y,z)}{x^2+y^2\leqslant 1,\ z=0},\\ \cS_2&= \Set{(x,y,z)}{z\ge0,\ x^2+y^2+z^2=1}, \end{align*}
    avec un vecteur normal unitaire pointant vers le haut, ont le cercle \(\cC\) pour bord. Ainsi, le théorème de Stokes dit que \(\iint_{\cS_1}\vnabla\times\vF\cdot\vn\ \dee{S} =\iint_{\cS_2}\vnabla\times\vF\cdot\vn\ \dee{S}\text{.}\)
  • Une seconde remarque concerne les champs \(\vF\) tels que \(\vnabla\times\vF=\vZero\) partout. Alors, le théorème de Stokes donne \(\oint_{\cC}\vF\cdot \dee{\vr}=0\) le long de toute courbe fermée \(\cC\text{,}\) ce qui entraîne que \(\vF\) est conservatif, en vertu du Théorème 4.4.7. Ainsi, le théorème de Stokes fournit une autre preuve du Théorème 4.4.9.
Voyons un premier exemple simple qui illustre comment le théorème de Stokes peut être utile lorsque \(\vnabla\times\vF\) peut être simplifié.

Exemple 4.8.2.

Calculer \(\displaystyle \oint_{\cC}\vF\cdot\dee{\vr} \text{,}\)\(\vF= \big[2z+\sin\big(x^{146}\big)\big]\,\vi-5z\,\vj -5y\,\vk\) et où la courbe \(\cC\) est le cercle \(x^2+y^2=4\) du plan \(z=1\text{,}\) orienté dans le sens antihoraire lorsque vu d’en haut.
Solution.
Le terme \(x^{146}\) dans \(\vF\) va vraisemblablement faire que l’évaluation directe de l’intégrale de ligne sera très difficile. Afin d’utiliser le théorème de Stokes, nous avons besoin d’une surface \(\cS\) telle que \(\partial S=C\text{.}\) Le choix le plus simple est le disque
\begin{equation*} S = \Set{(x,y,z)}{ x^2+y^2\leqslant 4,\ \ z=1}\text{.} \end{equation*}
Puisque
\begin{align*} \vnabla\times\vF & = \det\left[\begin{matrix} \vi & \vj & \vk \\ \frac{\partial }{\partial x} & \frac{\partial }{\partial y} & \frac{\partial }{\partial z} \\ 2z+\sin\big(x^{146}\big) & -5z & -5y \end{matrix}\right]\\ & =\vi \det\left[\begin{matrix} \frac{\partial }{\partial y} & \frac{\partial }{\partial z} \\ -5z & -5y \end{matrix}\right] -\vj\det\left[\begin{matrix} \frac{\partial }{\partial x} & \frac{\partial }{\partial z} \\ 2z+\sin\big(x^{146}\big) & -5y \end{matrix}\right]\\ &\hskip1in +\vk\det\left[\begin{matrix} \frac{\partial }{\partial x} & \frac{\partial }{\partial y} \\ 2z+\sin\big(x^{146}\big) & -5z \end{matrix}\right]\\ &=2\vj \end{align*}
et que le vecteur normal unitaire à \(\cS\) est \(\vk\text{,}\) le théorème de Stokes donne
\begin{align*} \oint_{\cC}\vF\cdot\dee{\vr} & = \iint_{\cS} \vnabla\times\vF\cdot\vn\,\dee{S} = \iint_{\cS} (2\vj)\cdot\vk\,\dee{S} = 0\text{.} \end{align*}
Reprenons l’exemple précédent avec une courbe un peu plus compliquée.

Exemple 4.8.3.

Calculer \(\displaystyle \oint_{\cC}\vF\cdot\dee{\vr}\text{,}\)\(\vF= \big[2z+\sin\big(x^{146}\big)\big]\,\vi-5z\,\vj -5y\,\vk\) et où \(\cC\) est l’intersection de \(x^2+y^2+z^2=4\) et \(z=y\text{,}\) orientée dans le sens antihoraire lorsque vue d’en haut.
Solution.
La surface \(x^2+y^2+z^2=4\) est la sphère de rayon \(2\) centrée à l’origine, et \(z=y\) est un plan qui contient l’origine. La courbe \(\cC\) est donc un cercle centré à l’origine et de rayon \(2\text{.}\) La portion du cercle qui se trouve dans le premier octant est esquissée dans la figure ci-après.
De nouveau, afin d’utiliser le théorème de Stokes, nous avons besoin d’une surface admettant \(\cC\) comme bord. Le choix le plus simple semble être le disque
\begin{equation*} S = \Set{(x,y,z)}{ x^2+y^2+z^2\leqslant 4,\ \ z=y}\text{.} \end{equation*}
La portion de \(\cS\) du premier octant est montrée ci-dessus. Nous avons vu dans l’Exemple 4.8.2 que
\begin{align*} \vnabla\times\vF &=2\vj\text{,} \end{align*}
de sorte que le théorème de Stokes donne
\begin{align*} \oint_{\cC}\vF\cdot\dee{\vr} & = \iint_{\cS} \vnabla\times\vF\cdot\vn\,\dee{S} = 2\iint_{\cS} \vj\cdot\vn\,\dee{S}\text{.} \end{align*}
Nous allons évaluer \(\displaystyle 2\iint_{\cS} \vj\cdot\vn\,\dee{S}\) de deux façons différentes. La première est plus efficace, mais nécessite un peu plus de perspicacité. Puisque \(\vnabla(z-y)=\vk-\vj\text{,}\) le vecteur normal unitaire pour le plan \(z-y=0\) (et donc pour \(\cS\)) est \(\vn = \frac{1}{\sqrt{2}}(\vk-\vj)\text{.}\) Ainsi, l’intégrande est
\begin{gather*} \vj\cdot\vn=\vj\cdot\Big(\frac{-\vj+\vk}{\sqrt{2}}\Big)=-\frac{1}{\sqrt{2}}\text{,} \end{gather*}
qui est une constante, et nous n’avons pas besoin de calculer \(\vn\,dS\) :
\begin{align*} \oint_{\cC}\vF\cdot\dee{\vr} & = 2\iint_{\cS} \vj\cdot\vn\,\dee{S} =-\sqrt{2} \iint_{\cS} \dee{S} = -\sqrt{2}\text{Aire}(S) =-\sqrt{2}\pi\, 2^2\\ &=-4\sqrt{2}\pi\text{.} \end{align*}
Voici une autre façon, en faisant les calculs. Comme \(\cS\) est contenue dans le plan \(z=f(x,y)=y\text{,}\) nous avons
\begin{gather*} \vn\,dS = \pm\big(-f_x\,\vi-f_y\,\vj+\vk\big)\,\dee{x}\dee{y} = \pm (-\vj+\vk)\,\dee{x}\dee{y}\text{.} \end{gather*}
Afin d’obtenir le vecteur normal unitaire pointant vers le haut, nous prenons le signe \(+\text{,}\) de sorte que \(\vn\,dS= (-\vj+\vk)\,\dee{x}\dee{y}\text{.}\) Lorsque \((x,y,z)\) parcourt
\begin{align*} S &= \Set{(x,y,z)}{ x^2+y^2+z^2\leqslant 4,\ \ z=y}\\ &= \Set{(x,y,z)}{ x^2+2y^2\leqslant 4,\ \ z=y}\\ &= \Set{(x,y,z)}{ \tfrac{x^2}{4} +\tfrac{y^2}{2}\leqslant 1,\ \ z=y}\text{,} \end{align*}
\((x,y)\) parcourt le disque elliptique \(R=\Set{(x,y)}{\frac{x^2}{4} +\frac{y^2}{2}\leqslant 1}\text{.}\) La portion de cette ellipse dans le premier octant est la région ombragée dans la figure ci-bas.
Cette ellipse a pour demi-axes \(a=2\) et \(b =\sqrt{2}\text{,}\) de sorte que son aire est \(\pi a b = 2\sqrt{2} \pi\text{.}\) Ainsi,
\begin{align*} \oint_{\cC}\vF\cdot\dee{\vr} & = 2\iint_{\cS} \vj\cdot\vn\,\dee{S} =2\iint_{\cR}\vj\cdot(-\vj+\vk)\,\dee{x}\dee{y} =-2 \iint_{\cR}\dee{x}\dee{y}\\ &= -2\text{Aire}(R)\\ &=-4\sqrt{2}\pi. \end{align*}
Dans l’exemple suivant nous calculons une intégrale de ligne de deux façons : comme une intégrale de ligne, puis grace à une intégrale double et le théorème de Stokes.

Exemple 4.8.4.

Calculer \(\displaystyle \int_{\cC}\vF\cdot\dee{\vr}\text{,}\)\(\vF= (x+y)\,\vi+2(x-z)\,\vj +(y^2+z)\,\vk\) et où \(\cC\) est la courbe orientée obtenue en allant de \((2,0,0)\) à \((0, 3, 0)\) puis à \((0, 0, 6)\) et en retournant à \((2, 0, 0)\text{,}\) le tout en suivant des segments de droite.
Solution 1.
Dans cette première solution, nous évaluons l’intégrale directement. La première portion (\(C_1\) dans la figure) peut être paramétrée par
\begin{equation*} \vr(t)= 2\vi + t(3\vj - 2 \vi) = (2-2t)\vi +3t\vj \qquad 0\leqslant t\leqslant 1\text{.} \end{equation*}
Ainsi, l’intégrale le long de cette première portion est
\begin{align*} \int_0^1 \vF(\vr(t))\cdot\diff{\vr}{t}\ \dee{t} &=\int_0^1 (2+t,\, 2(2-2t),\, (3t)^2)\cdot(-2,\, 3,\, 0)\ \dee{t}\\ &=\int_0^1 (8-14t)\ \dee{t}\\ &=\Big[8t-7t^2\Big]_0^1=1\text{.} \end{align*}
La deuxième courbe (\(C_2\) dans la figure) peut être paramétrée par
\begin{equation*} \vr(t)=3\vj + t(6\vk - 3 \vj) = (3-3t)\vj + 6t\vk \qquad 0\leqslant t\leqslant 1\text{.} \end{equation*}
Ainsi, l’intégrale le long de cette portion est
\begin{align*} \int_0^1 \vF(\vr(t))\cdot\diff{\vr}{t}\ \dee{t} &=\int_0^1 \big(3(1-t),\, - 12t,\, 9(1-t)^2+ 6t\big)\cdot(0, -3, 6)\ \dee{t}\\ &=\int_0^1 [36t+54(1-t)^2+36t]\ \dee{t}\\ &=\Big[18t^2-18(1-t)^3+18t^2\Big]_0^1\\ &=54\text{.} \end{align*}
Le dernier segment (\(C_3\) dans la figure) peut être paramétré par
\begin{equation*} \vr(t)= 6\vk + t(2\vj - 6\vk) = 2t\vi + (6-6t)\vk \qquad 0\leqslant t\leqslant 1\text{.} \end{equation*}
Ainsi, l’intégrale de ligne le long de ce segment est
\begin{align*} \int_0^1 \vF(\vr(t))\cdot\diff{\vr}{t}\ \dee{t} &=\int_0^1 \big(2t,\, 4t - 12(1-t),\, 6(1-t)\big)\cdot(2, 0, -6)\ \dee{t}\\ &=\int_0^1 [4t-36(1-t)]\ \dee{t} =\Big[2t^2+18(1-t)^2\Big]_0^1 =-16\text{.} \end{align*}
Globalement, l’intégrale qui nous intéresse est donc
\begin{equation*} \oint_{\cC}\vF\cdot \dee{\vr}=1+54-16=39\text{.} \end{equation*}
Solution 2.
Cette fois, nous utilisons le théorème de Stokes. Le rotationnel de \(\vF\) est
\begin{align*} \vnabla\times\vF &=\det\left[\begin{matrix} \vi & \vj &\vk \\ \frac{\partial }{\partial x} & \frac{\partial }{\partial y} & \frac{\partial }{\partial z} \\ x+y & 2(x-z) & y^2+z \end{matrix} \right]\\ & =(2y+2)\vi-(0-0)\vj+(2-1)\vk\\ &=2(y+1)\vi+\vk\text{.} \end{align*}
La courbe \(\cC\) est un triangle contenu dans le plan \(\frac{x}{2}+\frac{y}{3}+\frac{z}{6}=1\text{.}\) De plus, \(\cC\) est la frontière de la surface \(\cS\) qui consiste en la portion du plan \(\frac{x}{2}+\frac{y}{3}+\frac{z}{6}=1\) vérifiant \(x\geqslant0\text{,}\) \(y\geqslant0\) et \(z\geqslant0\text{.}\) Nous pouvons écrire l’équation du plan sous la forme \(z=6-3x-2y\text{.}\) Pour cette surface,
\begin{equation*} \vn\ \dee{S}=(3\vi+2\vj+\vk)\,\dee{x}\,\dee{y}\text{,} \end{equation*}
en vertu de la Formule 4.7.2, et nous pouvons écrire
\begin{align*} S&=\Set{(x,y,z)}{x\ge0,\ y\ge0,\ z\ge0,\ z=6-3x-2y}\\ &=\Set{(x,y,z)}{x\ge0, y\ge0,\ 6-3x-2y\ge0,\ z=6-3x-2y}\text{.} \end{align*}
Lorsque \((x,y,z)\) parcourt \(\cS\text{,}\) \((x,y)\) parcourt le triangle
\begin{align*} R&=\Set{(x,y,z)}{x\ge0,\ y\ge0,\ 3x+2y\leqslant 6}\\ &=\Set{(x,y,z)}{x\ge0,\ 0\leqslant y \leqslant \tfrac{3}{2}(2-x)}\text{.} \end{align*}
Nous calculons alors
\begin{align*} \oint_{\cC}\vF\cdot \dee{\vr}&=\iint_{\cS}\vnabla\times\vF\cdot\vn\,\dee{S}\\ &=\iint_{\cR}[2(y+1)\vi+\vk]\cdot[3\vi+2\vj+\vk]\ \dee{x}\,\dee{y}\cr &=\iint_{\cR}[6y+7]\ \dee{x}\,\dee{y}\\ & =\int_0^3 \int_0^{{1\over 3}(6-2y)}\ [6y+7]\ \dee{x}\,\dee{y}\\ &=\int_0^3 \frac{1}{3}[6y+7][6-2y] \dee{y}\\\ & =\frac{1}{3}\int_0^3 \dee{y}\ [-12y^2+22y+42]\\ &=\frac{1}{3}\Big[-4y^3+11y^2+42y\Big]_0^3\\ & =\big[-4\times 9 +11\times 3 +42\big] =39\text{.} \end{align*}
Voyons un exemple dans lequel notre attention sera dirigée surtout sur le champ à intégrer.

Exemple 4.8.5.

Évaluer \(\oint_{\cC}\vF\cdot\dee{\vr}\text{,}\)\(\vF= (\cos x +y+z)\,\vi+(x+z)\,\vj +(x+y)\,\vk\) et où \(\cC\) est l’intersection des surfaces
\begin{equation*} x^2+\frac{y^2}{2}+\frac{z^2}{3}=1\qquad\text{et}\qquad z=x^2+2y^2 \end{equation*}
orientée dans le sens antihoraire lorsque vue d’en haut.
Solution.
Pour commencer, faisons une esquisse de la courbe. La surface d’équation \(x^2+\frac{y^2}{2}+\frac{z^2}{3}=1\) est un ellipsoïde centré à l’origine, et \(z=x^2+2y^2\) est un paraboloïde qui s’ouvre vers le haut. Ils sont esquissés dans la figure ci-après.
Leur intersection, la courbe \(\cC\text{,}\) est la courbe bleue dans la figure, c’est une espèce de cercle déformé
 2 
Cherchez “La Persistance de la mémoire”, puis regardez la montre suspendue sur la branche de l’arbre.
.
On peut paramétrer \(\cC\text{,}\) par exemple en substituant \(x^2 = z - 2y^2\) dans l’équation de l’ellipsoïde, ce qui donne \(-\frac{3}{2}y^2 + \frac{1}{3}(z+\frac{3}{2}\big)^2 = \frac{7}{4}\text{.}\) On peut alors résoudre pour obtenir \(y\) comme fonction de \(z\text{,}\) puis \(x^2=z-2y^2\) donne aussi \(x\) en fonction de \(z\text{.}\) Cependant, ceci va mener, au mieux, à une intégrale pas très sympathique. Essayons plutôt le théorème de Stokes.
Nous avons
\begin{align*} \vnabla\times\vF &=\det\left[\begin{matrix}\vi&\vj&\vk \\ \frac{\partial }{\partial x} &\frac{\partial }{\partial y} &\frac{\partial }{\partial z} \\ \cos x + y+z&x+z&x+y\end{matrix}\right]\\ &=\vi\big(1-1\big)-\vj\big(1-1\big)+\vk\big(1-1\big)\\ &=\vZero\text{,} \end{align*}
de sorte que \(\vF\) est conservatif (en fait, \(\vF=\vnabla\big(\sin x + xy + xz +yz\big)\)). Comme \(\cC\) est une courbe fermée, \(\oint_{\cC}\vF\cdot\dee{\vr}=0 \text{.}\)

Exemple 4.8.6.

Dans cet exemple, on calcule de deux façons différentes l’intégrale de ligne \(\displaystyle \oint_{\cC}\vF\cdot \dee{\vr}\text{,}\)
\begin{equation*} \vF=(z-y)\,\vi-(x+z)\,\vj-(x+y)\,\vk \end{equation*}
et où \(\cC\) est la courbe \(x^2+y^2+z^2=4\text{,}\) \(z=y\) orientée dans le sens antihoraire lorsque vue d’en haut.
Solution 1.
Calcul direct :
Dans ce premier calcul, on utilise un paramétrage de \(\cC\text{.}\) Le plan \(z=y\) passe par l’origine, qui est le centre de la sphère \(x^2+y^2+z^2=4\text{.}\) Ainsi, \(\cC\) est un cercle qui, comme la sphère, a un rayon égal à \(2\) et est centré à l’origine. Nous paramétrons \(\cC\) par
\begin{equation*} \vr(t)=\vc+\rho\cos t\,\vi'+ \rho\sin t\,\vj' \qquad 0\leqslant t\leqslant 2\pi\text{,} \end{equation*}
  • \(\vc=(0,0,0)\) est le centre de \(\cC\text{,}\)
  • \(\rho=2\) est le rayon \(\cC\text{,}\) et
  • \(\vi'\) et \(\vj'\) sont deux vecteurs
    1. unitaires,
    2. parallèles au plan \(z=y\text{,}\) et
    3. orthogonaux.
La partie la plus difficile est de trouver ces vecteurs \(\vi'\) et \(\vj'\text{.}\)
  • Le point \((2,0,0)\) vérifie \(x^2+y^2+z^2=4\) et \(z=y\text{,}\) de sorte qu’il se trouve sur \(\cC\text{.}\) Nous pouvons choisir \(\vi'\) comme un vecteur unitaire parallèle au vecteur joignant le centre du cercle \((0,0,0)\) à \((2,0,0)\text{,}\) c’est-à-dire \(\vi'=(1,0,0)\text{.}\)
  • Puisque le plan contenant le cercle est \(z-y=0\text{,}\) le vecteur \(\vnabla(z-y)=(0,-1,1)\) est perpendiculaire à ce plan. Ainsi, \(\vk'=\frac{1}{\sqrt{2}}(0,-1,1)\) est un vecteur normal unitaire pour le plan \(z=y\text{.}\) Ainsi, \(\vj'=\vk'\times\vi'=\frac{1}{\sqrt{2}}(0,-1,1)\times(1,0,0) = \frac{1}{\sqrt{2}}(0,1,1)\) est un vecteur qui est orthogonal à \(\vi'\) et à \(\vk'\text{.}\) Comme \(\vj'\) est orthogonal à \(\vk'\text{,}\) il est parallèle au plan \(z=y\text{.}\)
En remplaçant dans \(\vc=(0,0,0)\text{,}\) \(\rho=2\text{,}\) \(\vi'=(1,0,0)\) et \(\vj'=\frac{1}{\sqrt{2}}(0,1,1)\text{,}\) on trouve
\begin{align*} &\vr(t)=2\cos t\,(1,0,0)+ 2\sin t\,\frac{1}{\sqrt{2}}(0,1,1) =2\Big(\cos t, \frac{\sin t}{\sqrt{2}},\frac{\sin t}{\sqrt{2}}\Big)\\ &0\leqslant t\leqslant 2\pi\text{.} \end{align*}
Afin de vérifier que ce paramétrage est correct, notons que \(x=2\cos t\text{,}\) \(y=\sqrt{2}\sin t\text{,}\) \(z=\sqrt{2}\sin t\) vérifie \(x^2+y^2+z^2=4\) et \(z=y\text{.}\)
À \(t=0\text{,}\) \(\vr(0)=(2,0,0)\text{.}\) Lorsque \(t\) croît, \(z(t)=\sqrt{2}\sin t\) croît aussi, et \(\vr(t)\) bouge vers le haut, vers \(\vr\big(\frac{\pi}{2}\big)=(0,\sqrt{2},\sqrt{2})\text{.}\) Ceci donne donc la bonne orientation antihoraire. Nous pouvons maintenant intégrer.
\begin{align*} \vr(t)&=\big(2\cos t, \sqrt{2}\sin t,\sqrt{2}\sin t\big)\cr \vr\,'(t)&=\big(-2\sin t,\sqrt{2}\cos t,\sqrt{2}\cos t\big)\cr \vF\big(\vr(t)\big)&=\big(z(t)-y(t),-x(t)-z(t),-x(t)-y(t)\big)\cr &=\big(\sqrt{2}\sin t-\sqrt{2}\sin t, -2\cos t-\sqrt{2}\sin t,-2\cos t-\sqrt{2}\sin t\big)\cr &=-\big(0, 2\cos t+\sqrt{2}\sin t,2\cos t+\sqrt{2}\sin t\big)\cr \vF\big(\vr(t)\big)\cdot \vr\,'(t) &=-\big[4\sqrt{2}\cos^2 t+4\cos t\sin t\big]\\ &=-\big[2\sqrt{2}\cos(2t)+2\sqrt{2}+2\sin(2t)\big] \end{align*}
En vertu des formules des angles doubles, \(\sin(2t)=2\sin t\,\cos t\) et \(\cos(2t) = 2\cos^2t-1\text{.}\) Ainsi,
\begin{align*} \oint_{\cC}\vF\cdot \dee{\vr} &=\int_0^{2\pi}\vF\big(\vr(t)\big)\cdot \vr\,'(t)\ \dee{t}\\ &=\int_0^{2\pi}-\big[2\sqrt{2}\cos(2t)+2\sqrt{2}+2\sin(2t)\big]\ \dee{t}\\ &=-\Big[\sqrt{2}\sin(2t)+2\sqrt{2}t-\cos(2t)\Big]_0^{2\pi}\\ &=-4\sqrt{2}\pi\text{.} \end{align*}
Voyons une autre façon de procéder.
Solution 2.
Théorème de Stokes
Afin d’utiliser le théorème de Stokes, nous devons exprimer \(\cC\) comme le bord \(\partial \cS\) d’une surface \(\cS\text{.}\) Puisque
\begin{equation*} \cC=\Set{(x,y,z)}{x^2+y^2+z^2=4,\ z=y} \end{equation*}
est une courbe fermée, ceci est possible. En fait, nous avons plusieurs choix pour \(\cS\text{.}\) Trois possibilités, esquissées ci-bas, sont
\begin{align*} S&=\Set{(x,y,z)}{x^2+y^2+z^2\leqslant 4,\ z=y},\cr S'&=\Set{(x,y,z)}{x^2+y^2+z^2= 4,\ z\geqslant y},\cr S''&=\Set{(x,y,z)}{x^2+y^2+z^2= 4,\ z\leqslant y}\text{.} \end{align*}
La première de ces surfaces, qui fait partie d’un plan, est la plus susceptible de donner lieu à des calculs simples.
Afin de nous préparer, calculons \(\vnabla\times\vF\) et \(\vn\,\dee{S}\text{.}\) Pour le dernier, nous appliquons la formule \(\vn\,\dee{S}=\pm(-f_x,-f_y,1)\,\dee{x}dy\) (de la Formule 4.7.2) à la surface \(z=f(x,y)=y\text{.}\) Nous choisissons le signe \(+\) pour avoir le vecteur normal pointant vers le haut.
\begin{align*} \vnabla\times\vF &=\det\left[\begin{matrix}\vi&\vj&\vk \\ \frac{\partial }{\partial x} &\frac{\partial }{\partial y} &\frac{\partial }{\partial z} \\ z-y&-x-z&-x-y\end{matrix}\right]\\ &=\vi\big(-1-(-1)\big)-\vj\big(-1-1\big)+\vk\big(-1-(-1)\big)\\ &=2\,\vj\\ \vn\,\dee{S}&=(0,-1,1)\,\dee{x}\dee{y}\\ \vnabla\times\vF\cdot\vn\,\dee{S}&=(0,2,0)\cdot(0,-1,1)\,\dee{x}\dee{y} =-2\,\dee{x}\dee{y} \end{align*}
Les variables d’intégration sont \(x\) et \(y\text{.}\) Par définition, le domaine d’intégration est
\begin{equation*} R=\Set{(x,y)}{(x,y,z)\text{ est dans }S\text{ pour un certain }z}\text{.} \end{equation*}
Afin de déterminer ce domaine plus précisément, observons que, puisque \(z=y\) sur \(\cS\text{,}\) \(x^2+y^2+z^2\leqslant 4\) équivaut à \(x^2+2y^2\leqslant 4\) sur \(\cS\) :
\begin{equation*} \cS=\Set{(x,y,z)}{x^2+2y^2\leqslant 4,\ z=y} \implies R=\Set{(x,y)}{x^2+2y^2\leqslant 4}\text{.} \end{equation*}
Ainsi, le domaine d’intégration est une ellipse avec semi-axe majeur \(a=2\) et semi-axe mineur \(b=\sqrt{2}\text{,}\) de sorte que son aire est \(\pi a b=2\sqrt{2}\pi\text{.}\) L’intégrale est alors
\begin{equation*} \oint_{\cC}\vF\cdot \dee{\vr} =\iint_{\cS} \vnabla\times\vF\cdot\vn\,\dee{S} =\iint_{\cR}(-2)\,\dee{x}\dee{y} =-2\ \text{Aire}\,(R) =-4\sqrt{2}\pi\text{.} \end{equation*}
Théorème de Stokes, prise 2
Puisque l’intégrande est une constante (après Stokage, pas l’intégrande original) et que \(\cS\) est si simple (nous avons fait de bons choix!), nous pouvons évaluer l’intégrale \(\iint_{\cS} \vnabla\times\vF\cdot\vn\,\dee{S}\) sans calculer \(\dee{S}\) explicitement et même établir les limites d’intégration. Nous avons \(\vnabla\times\vF=2\,\vj\text{.}\) Puisque \(\cS\) est la surface de niveau \(z-y=0\text{,}\) le gradient \(\vnabla(z-y)=-\vj+\vk\) est normal à \(\cS\text{.}\) Ainsi, \(\vn = \frac{1}{\sqrt{2}}(-\vj+\vk)\) et
\begin{align*} \oint_{\cC}\vF\cdot \dee{\vr} &=\iint_{\cS} \vnabla\times\vF\cdot\vn\,\dee{S} =\iint_{\cS} (2\vj)\cdot \frac{1}{\sqrt{2}}(-\vj+\vk)\,\dee{S}\\ &=\iint_{\cS} -\sqrt{2}\,\dee{S} =-\sqrt{2}\ {\rm Aire}\,(S)\text{.} \end{align*}
Puisque \(\cS\) est un cercle de rayon \(2\text{,}\) \(\oint_{\cC}\vF\cdot \dee{\vr}=-4\sqrt{2}\pi\text{,}\) encore une fois.
Voyons maintenant un exemple pour lequel il faut travailler un peu plus pour trouver la surface qui permet d’effectuer les calculs.

Exemple 4.8.7.

Soit
\begin{equation*} \vF = \left(z^2 - \cos(x^2) \right)\vi + \left(x^2 - \sin(y^2)\right)\vj + \left(xy - \tan(z^2)\right)\vk \end{equation*}
et \(\cC\) la courbe paramétrée par \(\vr(t) = 2\cos(t)\, \vi + 2 \sin t\, \vj + 4 \sin^2 t \, \vk\text{,}\) pour \(t\in [0,2\pi]\text{.}\) Calculer la valeur de \(\int_{\cC} \vF \cdot d\vr\text{.}\)
Solution.
Notons pour commencer que la courbe est fermée et simple. Le calcul de l’intégrale de ligne risque d’être ardu, en raison des termes \(\cos(x^2),\ \sin(y^2)\) et \(\tan(z^2)\text{.}\)
Afin d’utiliser le théorème de Stokes, il faudrait identifier une surface \(\cS\) telle que \(\partial \cS = \cC\text{.}\) En général, plusieurs choix sont possibles. Dans notre exemple, les composantes du chemin sont \(x(t) = 2\cos(t),\ y(t) = 2 \sin t\) et \(z(t) = 4 \sin^2 t\text{.}\) Ces composantes vérifient \(z(t)^2= y(t)^2\text{,}\) de sorte que notre courbe se trouve sur le cylindre parabolique \(z=y^2\text{.}\) Soit donc \(\cS\) la portion de ce cylindre qui est limitée par \(\cC\text{.}\)
Figure 4.8.8. La courbe \(\cC\) dans \(\R^3\) ainsi qu’une surface \(\cS\) dont elle est la frontière. La surface est orientée par le vecteur en bleu et elle est le graphe d’une fonction avec domaine \(\cR\text{,}\) en vert.
Le théorème de Stokes nous donne alors
\begin{equation*} \int_{\cC} \vF\cdot d\vr = \int_{\partial \cS} \vF\cdot d\vr = \iint_{\cS} (\vnabla \times \vF)\cdot d\vS. \end{equation*}
Calculons le rotationnel de notre champ
\begin{align*} \vnabla \times \vF \amp = \left| \begin{array}{ccc} \vi & \pdiff{}{x} & z^2 - \cos(x^2)\\ \vj & \pdiff{}{y} & x^2 - \sin(y^2)\\ \vk & \pdiff{}{z} & xy - \tan(z^2)\end{array} \right|\\ \amp = x\, \vi + (-y+2z)\vj + 2x\, \vk \end{align*}
La surface \(\cS\) est le graphe de la fonction \(f(x,y) = y^2\) avec \((x,y)\in \cR\text{,}\)\(\cR\) est définie par \(x^2+ y^2 \leqslant 2^2\text{.}\)
Le paramétrage donné de \(\cC\) lui impose une orientation. Lorsque vue d’en haut (avec le vecteur \(\vk\) pointant vers nous), \(\cC\) est parcourue dans le sens antihoraire. Le vecteur normal pour \(\cS\) qui est compatible avec cette orientation de \(\cC\) doit donc avoir une composante \(\vk\) positive. Nous choisissons donc le signe \(+\) dans la formule (✶✶) de notre inventaire de formules 4.7.12. Nous calculons alors
\begin{align*} \left(\vnabla \times \vF\right)\cdot \left(-f_x\, \vi - f_y\, \vj + \vk\right) \amp = \left(x\,\vi + (-y+2z)\vj + 2x\, \vk\right)\cdot\left(0\vi -2y\,\vj + \vk\right) \\ \amp = y^2 -4yz+2x = y^2 -4y^3 +2x \end{align*}
où, dans la dernière égalité, nous avons utilisé le fait que \(z = f(x,y) = y^2\text{.}\)
Ainsi, le flux qui nous intéresse vaut
\begin{align*} \iint_{\cR} 2y^2 -4y^3 +2x \, dA\amp = 2\iint_{\cR}y^2\, dA +4 \iint_{\cR}y^3 \, dA + 2\iint_{\cR} x\, dA \end{align*}
La symétrie de \(\cR\) et le fait que les intégrandes des deuxième et troisième intégrales sont des fonctions impaires font en sorte que ces deux intégrales sont nulles. Afin de calculer celle qui compte, nous utilisons les coordonnées polaires pour obtenir
\begin{align*} \int_{\cC} \vF \cdot d\vr \amp= \iint_{\cS} \vnabla\times \vF \cdot d\vS = 2\iint_{\cR} y^2 \, dA\\ \amp = 2\int_0^{2\pi}\int_0^2 r^2(\sin^2 \theta)\, r\, dr\, d\theta \\ \amp =2 \left(\int_0^{2\pi}\sin^2 \theta\, d\theta \right)\left(\int_0^2 r^3 \, dr\right) \\ \amp = 2 \left(\int_0^{2\pi} \frac{1}{2}(1-\cos 2\theta)\ d\theta\right)\left(\int_0^2 r^3 \, dr\right) = 2\pi\cdot \frac{16}{4} = 8\pi \end{align*}

Sous-section Application : la loi de Faraday

L’induction magnétique est un processus physique par lequel une différence de potentiel (un voltage) est induite par un champ magnétique qui varie dans le temps. Ce processus est exploité dans de nombreuses applications, notamment les générateurs électriques, les moteurs à induction, la cuisson par induction, le soudage par induction et le chargement par induction. On attribue généralement à Michael Faraday
 3 
Michael Faraday (1791-1867) était un physicien et chimiste anglais. Il est devenu un scientifique extrêmement influent bien qu’il n’ait reçu qu’une éducation formelle des plus élémentaires.
la découverte de l’induction magnétique. La loi de Faraday est la suivante. Soit \(\cS\) une surface orientée de bord \(\cC\text{.}\) Soit \(\vE\) et \(\vB\) les champs électrique et magnétique (qui varient avec le temps). On pose alors
\begin{align*} \oint_{\cC}\vE\cdot \dee{\vr}&=\text{tension le long de }\cC,\\ \iint_{\cS}\vB\cdot \vn \,\dee{S}&=\text{flux magnétique à travers }\cS. \end{align*}
Le voltage le long de \(\cC\) est l’opposé du taux de changement du flux magnétique à travers \(\cS\) :
\begin{equation*} \oint({\cC}\vE\cdot \dee{\vr}=-\frac{\partial }{\partial t}\iint_{\cS}\vB\cdot\vn \,\dee{S}. \end{equation*}
Nous pouvons reformuler ceci comme une équation aux dérivées partielles. En vertu du théorème de Stokes,
\begin{equation*} \oint_{\cC}\vE\cdot \dee{\vr}=\iint_{\cS}(\vnabla\times\vE)\cdot \vn\,\dee{S}, \end{equation*}
de sorte que la loi de Faraday devient
\begin{equation*} \iint_{\cS}\Big(\vnabla\times\vE+\frac{\partial\vB}{\partial t}\Big)\cdot\vn\,\dee{S}=0. \end{equation*}
Ceci étant vrai pour toutes les surfaces \(\cS\text{,}\) l’intégrande, si on le suppose continu, doit être nul.
En effet, soit \(\vG=\Big(\vnabla\times\vE+\frac{\partial\vB}{\partial t}\Big)\) et supposons qu’il existe un point \(\vx_0\) tel que \(\vG(\vx_0)\ne 0\text{.}\) Soit un vecteur unitaire \(\vn\) dans la direction \(\vG(\vx_0)\text{.}\) Soit en plus \(\cS\) un petit disque centré à \(\vx_0\) avec normal \(\vn\text{.}\) Alors, \(\vG(\vx_0)\cdot\vn \gt 0\) et, si nous choisissons notre disque suffisamment petit, la continuité de \(\vG\) donnera que \(\vG(\vx)\cdot\vn \gt 0\) pour tout \(\vx\) sur \(\cS\text{.}\) Alors, \(\iint_{\cS}\Big(\vnabla\times\vE+\frac{\partial\vB}{\partial t}\Big)\cdot \vn\,\dee{S} \gt 0\text{,}\) une contradiction.
Ainsi, \(\vG=\vZero\) partout, et l’on conclut que
\begin{equation*} \vnabla\times\vE+\frac{\partial\vB}{\partial t}=0. \end{equation*}
Il s’agit de l’une des équations de Maxwell.

Exercices Exercices

Pour se pratiquer.

1.
Chacune des figures ci-bas contient une esquisse d’une surface \(\cS\) et son bord \(\partial \cS\text{.}\) Le théorème de Stokes dit que \(\oint_{\partial \cS} \vF\cdot \dee{\vr} =\iint_{\cS} \vnabla \times \vF \cdot \vn\,\dee{S}\) lorsque \(\vn\) est un vecteur unitaire normal à \(\cS\) compatible avec l’orientation du bord. Dans chaque cas, esquisser un vecteur normal unitaire convenable.
(a)
(b)
(c)
Réponse.
(a)
(b)
(c)
2.
Soit
  • \(\cR\) une région bornée dans le plan \(xy\text{;}\)
  • Le bord \(\cC\) de \(\cR\) est formé d’une seule courbe fermée, simple et lisse par morceaux
    • qui est orienté de façon compatible avec l’orientation de \(\cR\text{,}\) c’est-à-dire de sorte que si l’on marche le long de \(\cC\) dans le sens donné, alors \(\cR\) est à gauche;
  • \(F_1(x,y)\) et \(F_2(x,y)\) ont des dérivées continues en chaque point de \(\cR\text{.}\)
Utiliser le théorème de Stokes pour montrer que
\begin{equation*} \oint_{C} \big[F_1(x,y)\,\dee{x} +F_2(x,y)\,\dee{y}\big] =\iint{\cR}\Big(\frac{\partial F_2}{\partial x} - \frac{\partial F_1}{\partial y}\Big)\ \dee{x}\dee{y}, \end{equation*}
c’est-à-dire pour montrer le théorème de Green.
Réponse.
See the solution.
3.
Montrer que \(\ \oint_\cC\phi\vnabla\psi\cdot \dee{\vr}=-\oint_\cC\psi\vnabla\phi\cdot \dee{\vr}\ \) pour toute paire de fonctions \(\phi\) et \(\psi\) qui ont des dérivées de premier ordre continues et toute courbe \(\cC\) qui est le bord d’une surface orientable lisse par morceaux.
Réponse.
See the solution.
4.
Soit \(\cC\) la courbe intersection du cylindre \(x^2+y^2=1\) avec la surface \(z=y^2\) orientée dans le sens contraire des aiguilles d’une montre lorsque vue d’en haut. Soit \(\vF=(x^2-y, \, y^2+x, \, 1)\text{.}\) Calculer \(\oint_{\cC}\vF\cdot \dee{\vr}\)
  1. par calcul direct;
  2. en utilisant le théorème de Stokes.
Réponse.
(a) \(2\pi\)
(b) \(2\pi\)
5.
Évaluer \(\oint_{\cC}\vF\cdot \dee{\vr}\text{,}\)\(\vF=ye^x\,\vi+(x+e^x)\,\vj+z^2\,\vk\) et \(\cC\) est la courbe
\begin{equation*} \vr(t)=(1+\cos t)\,\vi+(1+\sin t)\,\vj+(1-\sin t-\cos t)\,\vk, \qquad 0\leqslant t\leqslant 2\pi. \end{equation*}
Réponse.
\(\pi\)
6.
Trouver la valeur de \(\iint_{\cS} \vnabla \times \vF \cdot \vn\,\dee{S}\text{,}\)\(\vF = \big( z - y, \, x, \, -x \big)\) et \(\cS\) est l’hémisphère
\begin{equation*} \Set{(x, y, z) \in\R^3 }{ x^2 + y^2 + z^2 = 4,\ z \geqslant 0} \end{equation*}
orienté de sorte que le vecteur normal unitaire pointe en s’éloignant du centre de l’hémisphère.
Réponse.
\(8\pi\)
7.
Soit \(\cS\) la portion de la surface \(z=16-{(x^2+y^2)}^2\) qui se trouve sur le plan \(xy\text{.}\) Soit \(\vF = x\ln(1+z) \, \vi + x(3+y) \, \vj + y\cos z \, \vk \text{.}\) Calculer
\begin{equation*} \iint_{\cS}\vnabla\times\vF\cdot\vn\,\dee{S}, \end{equation*}
\(\vn\) est le vecteur normal à \(\cS\) pointant vers le haut.
Réponse.
\(12\pi\)
8.
Soit \(\cC\) l’intersection du paraboloïde \(z=4-x^2-y^2\) et du cylindre \(x^2+(y-1)^2=1\text{,}\) orientée dans le sens contraire des aiguilles d’une montre lorsque vue d’en haut. Soit \(\vF=xz\,\vi+x\,\vj+yz\,\vk\text{.}\) Trouver \(\oint_{\cC}\vF\cdot \dee{\vr}\text{.}\)
Réponse.
\(\pi\)
9.
Soit \(\vF = - ye^z\,\vi + x^3\cos z\,\vj + z\sin(xy)\,\vk\) et \(\cS\) la portion de la surface \(z = (1-x^2)(1-y^2)\) qui se trouve au-dessus du carré \(-1\leqslant x\leqslant 1\text{,}\) \(-1\leqslant y\leqslant 1\) du plan \(xy\text{.}\) Trouver le flux de \(\vnabla\times \vF\) vers le haut à travers \(\cS\text{.}\)
Réponse.
\(8\)
10.
Calculer l’intégrale \(\oint_{\cC}\vF\cdot \dee{\vr}\text{,}\) avec \(\vF = (e^{x^2} - yz, \, \sin y - yz, \, xz + 2y)\) et \(\cC\) le parcours triangulaire de \((1, 0, 0)\) à \((0, 1, 0)\) puis à \((0, 0, 1)\) et qui retourne à \((1, 0, 0)\text{.}\)
Réponse.
\(1\)
11.
Soit \(\vF(x,y,z)=-z\,\vi+x\,\vj+y\,\vk\) un champ de vecteurs. Utiliser le théorème de Stokes pour calculer \(\oint_{\cC}\vF\cdot \dee{\vr},\)\(\cC\) est l’intersection du plan \(z=y\) et de l’ellipsoïde \(\frac{x^2}{4}+\frac{y^2}{2}+\frac{z^2}{2}=1\text{,}\) orientée dans le sens contraire des aiguilles d’une montre lorsque vue d’en-haut.
Réponse.
\(4\pi\)
12.
Considérer le champ de vecteurs \(\vF(x,y, z) = z^2 \,\vi + x^2 \,\vj + y^2\,\vk\) dans \(\R^3\text{.}\)
  1. Calculer l’intégrale de ligne \(I_1 = \int_{C_1}\vF\cdot\dee{\vr}\) lorsque \(C_1\) est la courbe formée par les segments de droites \(L_1\) de \((2, 0, 0)\) à \((0, 2, 0)\text{,}\) \(L_2\) de \((0, 2, 0)\) à \((0, 0, 2)\) et finalement \(L_3\) de \((0, 0, 2)\) à \((2, 0, 0)\text{.}\)
  2. Une courbe fermée simple \(C_2\) sur plan \(E\,:\, x + y + z = 2\text{,}\) qui délimite une région \(\cR\) de ce plan dont l’aire est \(3\) et est orientée dans le sens contraire aux aiguilles d’une montre lorsque vue de la direction du demi-axe \(x\) positif. Calculer l’intégrale \(I_2 = \int_{C_2}\vF\cdot\dee{\vr}\text{.}\)
Réponse.
(a) \(8\)
(b) \(4\sqrt{3}\)
13.
Soit \(\cC = \cC_1 + \cC_2 + \cC_3\) la courbe donnée par l’union des courbes paramétrées
\begin{alignat*}{2} \vr_1(t) &= \big(2\cos t, 2\sin t, 0\big), &\qquad &0 \leqslant t \leqslant \pi/2,\\ \vr_2(t) &= \big(0, 2\cos t, 2\sin t\big), & &0 \leqslant t \leqslant \pi/2,\\ \vr_3(t) &= \big(2\sin t, 0, 2\cos t\big), & &0 \leqslant t \leqslant \pi/2. \end{alignat*}
  1. Faire une esquisse de \(\cC\) montrant chacune des portions \(\cC_1\text{,}\) \(\cC_2\text{,}\) et \(\cC_3\) en indiquant les orientations induites par les paramétrages donnés.
  2. Trouver et paramétrer une surface orientée \(\cS\) dont le bord est \(\cC\) (avec les orientations données).
  3. Calculer l’intégrale \(\int_{\cC} \vF\cdot\dee{\vr}\text{,}\)
    \begin{equation*} \vF = \Big(y + \sin(x^2), \, z - 3x + \ln(1 + y^2), \, y + e^{z^2}\Big). \end{equation*}
Réponse.
(a)
(b) \(S=\Set{(x,y,z)}{x^2+y^2+z^2=4,\ x\geqslant 0,\ y\geqslant 0,\ z\geqslant 0}\) avec
\begin{align*} &\vr(\theta,\varphi) = 2\cos\theta\sin\varphi\ \vi + 2\sin\theta\sin\varphi\ \vj + 2\cos\varphi\ \vk,\\ &0\le\theta\le\frac{\pi}{2},\ 0\le\varphi\le\frac{\pi}{2} \end{align*}
et
\begin{equation*} \vn = \cos\theta\sin\varphi\ \vi +\sin\theta\sin\varphi\ \vj +\cos\varphi\ \vk =\frac{1}{2}\,\vr(\theta,\varphi) \end{equation*}
(c) \(-4\pi\)
14.
On considère le cône \(z = \sqrt{x^2 + y^2}\text{.}\) Noter que son sommet se trouve à l’origine \((0, 0, 0)\text{.}\) Le cône est orienté de sorte que, en chaque point, le vecteur normal pointe vers le bas (et en s’éloignant de l’axe des \(z\)). Dans les questions qui suivent, tant \(\cS_1\) que \(\cS_2\) sont orientées de la sorte.
Soit \(\vF = \big(-zy, \, zx, \, xy \cos(yz)\big)\text{.}\)
  1. Soit \(\cS_1\) la portion du cône qui se trouve entre les plans \(z = 0\) et \(z = 4\text{.}\) Noter que \(\cS_1\) n’inclut aucune portion du plan \(z = 4\text{.}\) Utiliser le théorème de Stokes pour calculer la valeur de
    \begin{equation*} \iint_{S_1} \vnabla\times\vF \cdot \vn\,\dee{S}. \end{equation*}
    Faire une esquisse inscrivant les orientations de \(\cS_1\) ainsi que du ou des contours d’intégration.
  2. Soit \(\cS_2\) la portion du cône qui se trouve au-dessous du plan \(z = 4\) et au-dessus du plan \(z = 1\text{.}\) Noter que \(\cS_2\) n’inclut aucune région des plans \(z = 1\) ou \(z = 4\text{.}\) Déterminer le flux de \(\vnabla\times\vF\) à travers \(\cS_2\text{.}\) Justifier en incluant une esquisse de \(\cS_2\) qui inclut son orientation ainsi que celle des contours d’intégration.
Réponse.
(a) \(-128\pi\)
(b) \(-126\pi\)
15.
Soit \(\cC\) la courbe d’intersection du plan \(z = x + 4\) et du cylindre \(x^2 + y^2 = 4\text{,}\) parcourue dans le sens des aiguilles d’une montre lorsque vue d’au-dessus.
Soit \(\vF(x, y, z) = \big(x^3 + 2y, \, \sin(y) + z, \, x + \sin(z^2)\big)\text{.}\)
Utiliser le théorème de Stokes pour évaluer l’intégrale de ligne \(\oint_{\cC}\vF\cdot\dee{\vr}\text{.}\)
Réponse.
\(4\pi\)
16.
Calculer l’intégrale
\begin{equation*} \int_{\cC}\left(z+\frac{1}{1+z}\right)\dee{x} +xz\,\dee{y} +\left(3xy-\frac{x}{(z+1)^2}\right)\dee{z}, \end{equation*}
\(\cC\) est la courbe paramétrée par
\begin{equation*} \vr(t) = \big( \cos t, \, \sin t, \, 1 - \cos^2 t \sin t \big), \qquad 0 \leqslant t \leqslant 2\pi. \end{equation*}
Réponse.
\(5\pi/4\)
17.
Une courbe fermée, simple et lisse par morceaux \(\cC\) se trouve sur le plan \(x + y + z = 1\text{.}\) La région du plan que la courbe \(\cC\) délimite a une aire égale à \(5\text{.}\) La courbe \(\cC\) est orientée dans le sens des aiguilles d’une montre lorsque vue du demi-axe positif des \(z\text{.}\)
Calculer la circulation de \(\vF (x, y, z) = (z^2, \, x^2, \, y^2)\) autour de \(\cC\text{.}\)
Réponse.
\(-\frac{10}{\sqrt{3}}\)
18.
Soit \(\cC\) la courbe fermée formée par les segments de droites allant de \((0, 0, 0)\) à \((0, 1, 1)\text{,}\) de \((0, 1, 1)\) à \((0, 1, 2)\text{,}\) de \((0, 1, 2)\) à \((0, 2, 0)\text{,}\) de \((0, 2, 0)\) à \((2, 2, 0)\) et de \((2, 2, 0)\) à \((0, 0, 0)\text{.}\) Soit
\begin{equation*} \vF = (-y+e^x\sin x)\,\vi +y^4\,\vj +\sqrt{z}\tan z\,\vk. \end{equation*}
Calculer l’intégrale \(\int_{\cC}\vF\cdot\dee{\vr}\text{.}\)
Réponse.
\(-2\)
19.
Soit \(\cC\) l’intersection du cylindre \(x^2 + y^2 = 1\) avec la surface \(z = xy^2\text{,}\) parcourue dans le sens des aiguilles d’une montre lorsque vue d’en haut. Calculer
\begin{equation*} \int_{\cC} (z + \sin z) \,\dee{x} + (x^3 - x^2 y) \,\dee{y} + (x \cos z - y) \,\dee{z}. \end{equation*}
Réponse.
\(-\pi\)
20.
Calculer \(\iint_{\cS} \vnabla\times\vF\cdot\vn\,\dee{S}\text{,}\)\(\cS\) est la portion de la sphère \(x^2+y^2+z^2=2\) au-dessus du plan \(z=1\text{,}\) \(\vn\) pointe vers le haut et
\begin{gather*} \vF(x,y,z) = -y^2\,\vi +x^3\,\vj + \big(e^x + e^y +z\big)\,\vk. \end{gather*}
Réponse.
\(\frac{3\pi}{4}\)
21.
Soit \(\vF = x \sin y\,\vi - y \sin x\,\vj + (x - y)z^2\,\vk \text{.}\) Au moyen du théorème de Stokes, calculer la valeur de
\begin{equation*} \int_{\cC}\vF\cdot \dee{\vr} \end{equation*}
le long du chemin formé par les segments de droites joignant successivement les points \(P_0 = (0, 0, 0)\text{,}\) \(P_1 = (\pi/2, 0, 0)\text{,}\) \(P_2 = (\pi/2, 0, 1)\text{,}\) \(P_3 = (0, 0, 1)\text{,}\) \(P_4 = (0, \pi/2, 1)\) et \(P_5 = (0, \pi/2, 0)\text{,}\) puis de retour à \((0, 0, 0)\text{.}\)
Réponse.
\(\frac{\pi}{3}\)
22.
Soit
\begin{equation*} \vF=\left( \frac{2z}{1+y}+\sin(x^2), \, \frac{3z}{1+x}+\sin(y^2), \, 5(x+1)(y+2) \right) \end{equation*}
et \(\cC\) la courbe orientée formée des segments de droites joignant successivement les points \((0,0,0)\text{,}\) \((2,0,0)\text{,}\) \((0,0,2)\text{,}\) \((0,3,0)\) et de retour à \((0,0,0)\text{.}\)
  1. Faire une figure montrant \(\cC\) et l’orientation de chaque segment.
  2. Calculer la valeur de \(\int_{\cC}\vF\cdot\dee{\vr}\text{.}\)
Réponse.
(a)
(b) \(\int_{\cC}\vF\cdot\dee{\vr} = 10\)
23.
Calculer \(\displaystyle\iint_{\cS}\vnabla\times\vF\cdot\vn\,\dee{S}\text{,}\)\(\vF=y\,\vi+2z\,\vj+3x\,\vk\) et \(\cS\) est la surface \(z=\sqrt{1-x^2-y^2}\text{,}\) \(z\geqslant 0\) orientée par un vecteur normal unitaire \(\vn\) vérifiant \(\vn\cdot\vk\geqslant 0\text{.}\)
Réponse.
\(-\pi\)
24.
Soit \(\cS\) la surface
\begin{equation*} x^2 + y^2 + 2(z-1)^2 = 6, \quad z \geqslant 0 \end{equation*}
orientée par le vecteur normal unitaire qui pointe vers l’extérieur (par exemple, au point le plus haut de la surface, le vecteur normal est \(\vk\)).
Soit
\begin{equation*} \vG = \nabla\times\vF, \quad\text{où}\quad \vF = (xz - y^3\cos z)\,\vi + x^3 e^z\,\vj + xyze^{x^2+y^2+z^2}\,\vk. \end{equation*}
Calculer \(\displaystyle \iint_\cS \vG\cdot \vn\dee{S}\text{.}\)
Réponse.
\(24\pi\)
25.
Soit \(\cC\) un cercle de rayon \(R\) contenu dans le plan \(x+y+z=3\text{.}\) Utiliser le théorème de Stokes pour calculer
\begin{equation*} \oint_{\cC}\vF\cdot \dee{\vr}. \end{equation*}
\(\vF=z^2\vi+x^2\vj+y^2\vk\) (utiliser n’importe quelle orientation du cercle).
Réponse.
\(2\sqrt{3}\pi R^2\)
26.
Soit \(\cS\) la surface formée de la partie supérieure et des 4 faces latérales du cube de sommets \((\pm 1, \, \pm 1, \, \pm 1)\text{,}\) orientée vers l’extérieur du cube. Si \(\vF(x,y,z)=(xyz,xy^2,x^2yz)\text{,}\) trouver le flux de \(\vnabla\times\vF\) à travers \(\cS\text{.}\)
Réponse.
\(\frac{4}{3}\)
27.
Soit \(\cS\) la portion de la rampe spirale (c’est une surface hélicoïdale) donnée par
\begin{equation*} x=u\cos v,\ y=u\sin v,\ z=v, \qquad 0\leqslant u\leqslant 1,\ 0\leqslant v\leqslant 2\pi. \end{equation*}
Soit \(\cC\) le bord de \(\cS\) avec l’orientation donnée par le vecteur normal pointant vers le haut. Trouver
\begin{equation*} \int_{\cC} y\,\dee{x}-x\,\dee{y}+ xy\,\dee{z}. \end{equation*}
Réponse.
\(-2\pi\)

Groupe d'exercices.

28.
Soit \(\cC\) l’intersection de \(x+2y-z=7\) et \(x^2-2x+4y^2=15\text{.}\) La courbe \(\cC\) est orientée dans le sens contraire des aiguilles d’une montre lorsque vue d’en haut. Soit
\begin{equation*} \vF=\big(e^{x^2}+yz\big)\,\vi +\big(\cos(y^2)-x^2\big)\,\vj +\big(\sin(z^2)+xy\big)\,\vk. \end{equation*}
Calculer \(\oint_{\cC}\vF\cdot \dee{\vr}\text{.}\)
Réponse.
\(24\pi\)
29.
  1. Calculer le rotationnel du champ \(\vF = \big(2 + x^2 + z, \, 0, \, 3 + x^2 z\big)\text{.}\)
  2. Soit \(\cC\) la courbe dans \(\R^3\) allant de \((0, 0, 0)\) au point \((2, 0, 0)\text{,}\) formée de trois segments de droites consécutifs : de \((0, 0, 0)\) à \((0, 0, 3)\text{,}\) de \((0, 0, 3)\) à \((0, 1, 0)\) et de \((0, 1, 0)\) à \((2, 0, 0)\text{.}\) Calculer
    \begin{equation*} \int_{\cC} \vF\cdot\dee{\vr}. \end{equation*}
    \(\vF\) est le champ de la partie (a).
Réponse.
(a) \(\vnabla\times\vF=(1-2xz)\,\vj\)
(b) \(\frac{20}{3}\)
30.
  1. Soit \(\cS\) la surface en forme de seau formée par la portion du cylindre \(y^2 + z^2 = 9\) comprise entre \(x = 0\) et \(x = 5\) ainsi que par le disque sur le plan \(yz\) de rayon \(3\) centré à l’origine (le seau \(\cS\) a une partie inférieure, mais pas de couvercle). Orienter \(\cS\) de sorte que le vecteur normal pointe vers l’extérieur. Calculer le flux de \(\vnabla\times\vG\) à travers \(\cS\text{,}\) lorsque \(\vG = (x, -z, y)\text{.}\)
  2. Calculer le flux de \(\vF = (2 + z, \, xz^2, \, x \cos y) \) à travers \(\cS\text{,}\)\(\cS\) est comme dans la partie (a).
Réponse.
(a) \(-18\pi\)
(b) \(-18\pi\)
31.
Soit
\begin{equation*} \vF(x, y, z) = \Big( \frac{y}{x} +x^{1+x^2}, \, x^2-y^{1+y^2}, \, \cos^5(\ln z)\Big). \end{equation*}
  1. Donner le domaine \(\cD\) de \(\vF\text{.}\)
  2. Encercler les énoncés qui sont vrais :
    1. \(\cD\) est connexe.
    2. \(\cD\) est simplement connexe.
    3. \(\cD\) est disconnexe.
  3. Calculer \(\vnabla\times\vF\text{.}\)
  4. Soit \(\cC\) le carré dont les sommets sont \((3 \pm 1, 3 \pm 1, 2)\) orienté dans le sens des aiguilles d’une montre lorsque vu d’en bas. Calculer
    \begin{equation*} \int_{\cC} \vF \cdot \dee{\vr}. \end{equation*}
  5. Le champ \(\vF\) est-il conservatif?
Réponse.
(a) \(D=\Set{(x,y,z)}{x \gt 0,\ y \gt 0,\ z \gt 0}\)
(b) Le domaine \(\cD\) est connexe et simplement connexe.
(c) \(\vnabla\times\vF=\big(2x-\frac{1}{x}\big)\vk\)
(d) \(2\ln 2-24\)
(e) Non. \(\vF\) n’est pas conservatif.
32.
Une physicienne étudie un champ de vecteurs \(\vF(x, y, z)\text{.}\) Ses expériences permettent de conclure que \(\vF\) est de la forme
\begin{equation*} \vF(x, y, z) = xz\,\vi + (axe^y z + byz)\,\vj + (y^2 - xe^y z^2 )\,\vk \end{equation*}
pour certains nombres réels \(a\) et \(b\text{.}\) De plus, il est connu que \(\vF = \vnabla\times\vG\) pour un certain champ différentiable \(\vG\text{.}\)
  1. Trouver \(a\) et \(b\text{.}\)
  2. Calculer l’intégrale de surface
    \begin{equation*} \iint_{\cS}\vF\cdot\vn\,\dee{S}. \end{equation*}
    \(\cS\) est la portion de l’ellipsoïde \(x^2 + y^2 + \frac{1}{4} z^2 = 1\) telle que \(z \geqslant 0\text{,}\) orientée de sorte que le vecteur normal ait une composante \(z\) positive.
Réponse.
(a) \(a=2\text{,}\) \(b=-1\)
(b) \(\frac{\pi}{4}\)
33.
Soit \(\cC\) la courbe dans le plan \(xy\) allant du point \((0, 0)\) à \((5, 5)\) formée par les segments de droites reliant successivement les points \((0,0)\text{,}\) \((0,1)\text{,}\) \((1,1)\text{,}\) \((1,2)\text{,}\) \((2,2)\text{,}\) \((2,3)\text{,}\) \((3,3)\text{,}\) \((3,4)\text{,}\) \((4,4)\text{,}\) \((4,5)\) et \((5,5)\text{.}\) Calculer
\begin{gather*} \int_{\cC} \vF \cdot\dee{\vr}, \end{gather*}
\begin{gather*} \vF = y\,\vi + (2x - 10)\,\vj. \end{gather*}
Réponse.
\(-15\)
34.
Soit \(\vF = \big(\sin x^2, \, xz, \, z^2\big)\text{.}\) Calculer \(\oint_{\cC}\vF\cdot\dee{\vr}\) le long de la courbe \(\cC\) qui est l’intersection du cylindre \(x^2+y^2=4\) avec la surface \(z=x^2\text{,}\) parcourue dans le sens contraire des aiguilles d’une montre lorsque vue d’en haut.
Réponse.
\(12\pi\)
35.
Expliquer comment la loi de Faraday, sous sa forme différentielle
\begin{equation*} \vnabla\times\vE = -\frac{1}{c}\frac{\partial\vH}{\partial t}, \end{equation*}
est déduite de la forme intégrale
\begin{equation*} \oint_{\cC}\vE\cdot\dee{\vr} = -\frac{1}{c}\ \diff{ }{t}\iint_{\cS}\vH\cdot\vn\,\dee{S}. \end{equation*}
Réponse.
Réécrire \(\oint_{\cC}\vE\cdot\dee{\vr}\) comme une intégrale de surface. Voir la solution pour les détails.
36.
Soit \(\cC\) la courbe donnée par
\begin{equation*} x=\cos t,\ y=\sqrt{2}\sin t,\ z=\cos t,\ 0\leqslant t\leqslant 2\pi \end{equation*}
et soit
\begin{equation*} \vF=z\,\vi+x\,\vj+y^3z^3\,\vk. \end{equation*}
Utiliser le théorème de Stokes pour calculer
\begin{equation*} \oint_{\cC}\vF\cdot \dee{\vr}. \end{equation*}
Réponse.
\(\sqrt{2}\pi\)
37.
Utiliser le théorème de Stokes pour calculer
\begin{equation*} \oint_{\cC}z\,\dee{x}+x\,\dee{y}-y\,\dee{z} \end{equation*}
lorsque \(\cC\) est la courbe fermée donnée par l’intersection du plan \(x+y+z=1\) avec la sphère \(x^2+y^2+z^2=1\text{.}\) La courbe \(\cC\) est orientée dans le sens des aiguilles d’une montre lorsque vue de l’origine.
Réponse.
\(\frac{2\pi}{3\sqrt{3}}\)
38.
Soit \(\cS\) la portion du demi-cône
\begin{equation*} z=\sqrt{x^2+y^2},\quad y\geqslant 0, \end{equation*}
qui se trouve en dessous du plan \(z=1\text{.}\)
  1. Donner un paramétrage de \(\cS\text{.}\)
  2. Calculer le flux du champ
    \begin{equation*} \vv=x\,\vi + y\, \vj -2 z\,\vk \end{equation*}
    vers le bas à travers \(\cS\text{.}\)
  3. Un champ de vecteurs \(\vF\) a pour rotationnel \(\vnabla\times \vF= x\, \vi + y\, \vj -2 z\,\vk\text{.}\) Sur le plan \(xz\text{,}\) le champ est constant, avec \(\vF(x,0,z)=\vj\text{.}\) Avec cette information, calculer
    \begin{equation*} \int_\cC \vF\cdot \dee{\vr}, \end{equation*}
    \(\cC\) est le demi-cercle
    \begin{equation*} x^2+y^2=1,\ z=1,\ y\geqslant 0 \end{equation*}
    orienté de \((-1,0,1)\) à \((1,0,1)\text{.}\)
Réponse.
(a) Une paramétrisation possible est \(\vr(r, \theta) = r\cos \theta\, \vi +r\sin \theta\, \vj + r\,\vk\) avec \(0\leqslant r\leqslant 1\text{,}\) \(0\leqslant \theta \leqslant \pi\text{.}\)
(b) \(\pi\)
39.
Soit \(\iint_{\cS}(\vnabla\times\vF)\cdot\vn\,dS\text{,}\)\(\cS\) est la portion de la sphère \(x^2+y^2+z^2=1\) vérifiant \(x+y+z\geqslant 1\text{,}\) \(\vn\) est le vecteur normal qui pointe vers le haut et \(\vF=(y-z)\vi+(z-x)\vj+(x-y)\vk\text{.}\) Trouver une autre surface \(\cS'\) avec la propriété que \(\iint_{\cS}(\vnabla\times\vF)\cdot\vn\,\dee{S} =\iint_{\cS'}(\vnabla\times\vF)\cdot\vn\,\dee{S}\) et évaluer \(\iint_{\cS'}(\vnabla\times\vF)\cdot\vn\,\dee{S}\text{.}\)
Réponse.
\(-\frac{4}{\sqrt{3}}\pi\)